PSAT Reading Question 393: Answer and Explanation

Question: 393

The passage suggests that in order for a comprehensive physical theory to achieve widespread acceptance, it must have

  • A. both theoretical and philosophical consistency.
  • B. both mathematical and observational consistency.
  • C. both experimental and investigational consistency.
  • D. both standard and symmetrical consistency.

Correct Answer: B

Explanation:

(B) Lines 44–45 speak of an answer to this. They state that half of the battle in creating a physical theory is mathematical consistency and the other half is experimental evidence. So there will be both mathematical and observational (from experimental observations) consistency. Choice (A) is wrong because the passage does not mention philosophical consistency. Choice (C) is incorrect because investigational and experimental are synonymous. Choice (D) is not right because these terms focus on minor specific names and aspects of theories rather than on their general characteristics.

Test Information

  • Use your browser's back button to return to your test results.
  • Do more Reading tests.

More Tests

    All content of site and practice tests © 2022 Jack.
    Quick View

    PSAT Practice Tests

    More Information